Author Topic: EM Drive Developments - related to space flight applications - Thread 3  (Read 3131407 times)

Offline SeeShells

  • Senior Member
  • *****
  • Posts: 2442
  • Every action there's a reaction we try to grasp.
  • United States
  • Liked: 3186
  • Likes Given: 2708
doing frustum research in frankenmuth michigan today  :)
I see your checking out the disappearing effervescent waves. :D Thanks, needed a smile.

Shell

Offline SeeShells

  • Senior Member
  • *****
  • Posts: 2442
  • Every action there's a reaction we try to grasp.
  • United States
  • Liked: 3186
  • Likes Given: 2708
Thinking helps me to take a pause.

I have a silly question that I'm trying to wrap my head around it and need some help.
The group velocity holds the energy, momentum and information in a waveguide but the phase velocity can be superluminal in a waveguide. I see this.
But...
When the waves carrying energy, momentum and information reach cutoff and collapse in a wave guide then the superluminal phase velocity also collapses and in that instant where the phase velocities are collapsing from a superluminal speed into the group velocity wave front. What happens? Make sense?

Shell

Offline flux_capacitor

  • Full Member
  • ****
  • Posts: 708
  • France
  • Liked: 860
  • Likes Given: 1076
This short length, constant diameter, cylindrical step out ("retraction" size of the cover depth) design also seems to eliminate the need for spherical end plates, which would be a really big win.

Yes but the Chinese used a noisy 2500W magnetron (big oven, huh?) which has AM and FM modulation over a wider bandwidth than your cleaner solid-state 100W RF amp. I remember Shawyer told you to use a magnetron with flat end plates, but a narrow-band emitter with spherical ends.
« Last Edit: 07/26/2015 05:30 pm by flux_capacitor »

Offline X_RaY

  • Full Member
  • ****
  • Posts: 852
  • Germany
  • Liked: 1146
  • Likes Given: 2479
@SeeShells
sorry about your loss

Offline TheTraveller

This short length, constant diameter, cylindrical step out ("retraction" size of the cover depth) design also seems to eliminate the need for spherical end plates, which would be a really big win.

Yes but the Chinese used a noisy 1000W magnetron, which has AM and FM modulation over a wider bandwidth than your cleaner solid-state 100W RF amp. I remember Shawyer told you to use a magnetron with flat end plates, but a narrow-band emitter with spherical ends.

Correct.

But the new build Prof Yang has shared allows using flat end plates to obtain a Q of 117,500 by using short constant diameter set backs as attached. I assume the set backs convert a spherical wavefront into a planar wavefront in both directions.

When using a tapered waveguide to connect different diameter cylindrical waveguides this is what happens. Planar going in, spherical inside the tapered section and planar when emerging into the opposite side constant diameter cylindrical section.

I see this as a major advancement in EMDrive frustum design.

My 1st build has now altered to be able to experiment with this new structure as attached. Double ended tuning with sliding end plates inside constant diameter sections. Shawyer did do this for the small end in his Demonstrator EMDrive and repeated it with the Tajmar EMDrive. Both magnetron driven.

If this setup can eliminate the need for spherical end plates and still deliver an unloaded Q of 117,500, bring it on.

BTW to those "group velocity is different at each end of the frustum" doubters, wonder what the group velocity will be in those constant diameter sections? Would suggest it will be the group velocity as per the constant diameter circular waveguide equations. Which means Cullen, Shawyer & Prof Yang are correct.
« Last Edit: 07/26/2015 05:54 pm by TheTraveller »
It Is Time For The EmDrive To Come Out Of The Shadows

Offline X_RaY

  • Full Member
  • ****
  • Posts: 852
  • Germany
  • Liked: 1146
  • Likes Given: 2479
@WarpTech
was confused yesterday caused by the notation "kr". Dr Rodal posted the description of what is meaning it (namely not the radii of the cone at a given point at the z-axis, but the radius based on the cone apex).

@Rodal
Thanks for the explanation
http://forum.nasaspaceflight.com/index.php?topic=37642.msg1409751#msg1409751
« Last Edit: 07/26/2015 07:16 pm by X_RaY »

Offline WarpTech

  • Full Member
  • ****
  • Posts: 1407
  • Do it!
  • Statesville, NC
  • Liked: 1453
  • Likes Given: 1925
...
EDIT: I didn't understand kr was actually k * r
So it depends on frequency besides distance from the apex.
I agree, when I first saw the notation kr in Yang and Fan's paper I found that confusing too, as it could be interpreted as k sub r, a variable instead of two variables multiplied by each other.  I don't like the notation kr.  It would be better if the multiplication sign would be included or if big separation would be included to make it clear that it is the product of two variables, one of the variables (k) dependent on wavelength, and hence dependent on frequency.

I concur. Following this idea: since k = 2 Pi/wavelength

A small k implies a longer wavelength so a lower frequency.

A small k*r is dimensionless but implies both a low operating frequency and a short distance from the apex

In a closed cavity there are two ways of getting closer to the apex:
1- Increase the cone angle so the apex is closer to the small end plate
or
2- Make the end plate as smaller as possible

Since we saw Yang's design with a low cone angle is more efficient, we should not try (1) but (2). Which is difficult since the lower the frequency, the bigger the frustum.

Am I right on this?

But how the group velocity can become superluminal while it decreases is still beyond me.
It should be explored, but until a theory is corroborated to explain the anomalous thrust, what is optimal will not be known

EDIT: but yes, getting closer to the apex is interesting, that's why I wrote the paper about the fact that strong cut-off doesn't apply to conical cavities.  Getting close to the apex brings up General Relativity and other interesting things into the picture.  However, getting close to the apex may also affect the Q.  It needs to be tested

Look up the Reissner-Nordstrom metric: https://en.wikipedia.org/wiki/Reissner–Nordström_metric

(rQ/r)2 is Lorentz Force*G/c4

So it has to do with the EM pressure and stress near the apex. The group velocity of the wave slows down until it becomes an evanescent wave, and then it goes superluminal.

However, my hypothesis is in the other direction. Energy input at the small end that is "squeezed" by the cavity will expand at superluminal speed, and exert a much higher thrust than a photon rocket. Now, we have a metric equation to describe the effect, with numerical simulations by Zeng and Fan, as well as experimental evidence in vacuum that supports the effect. I think, theoretically, this is a giant leap in understanding of how it works. I need to crunch some data and see where each frustum falls on the impedance curve. That will tell us what the thrust-to-power ratio "could be" at the right k value.

NOTE: I am NOT saying that an EM drive can achieve FTL. No Way! SR will prevent it, but what I am saying is that the EM energy in the reference frame of the cavity, can exert forces greater than a photon rocket. This is not new physics, it is GR in the case of a large EM force and a small mass.

1 > (Q/r)2 > (2M/r)

Todd



Offline WarpTech

  • Full Member
  • ****
  • Posts: 1407
  • Do it!
  • Statesville, NC
  • Liked: 1453
  • Likes Given: 1925
@WarpTech
was confused yesterday caused by the notification "kr". Dr Rodal posted the description of what is meaning it (namely not the radii of the cone at a given point at the z-axis, but the radius based on the cone apex).

@Rodal
Thanks for the explanation
http://forum.nasaspaceflight.com/index.php?topic=37642.msg1409751#msg1409751

I'm not confused by this. I know that kr = k*r, is the distance from the cone apex in terms of phase. I agree the paper is confusing in the way it is written because they also use kr.

In my equation for the tapered cylinder using Bessel functions, I had z*w/c, which is the same thing. For a given k value based on the input frequency, k being the wavenumber of the propagating mode in the z direction, r is the distance from the apex. It is simply their way of normalizing the graph, because computing the Hankle functions is easier that way.

The graphs I posted for the Reissner-Nordstrom metric are equivalent representations for k=1, where r is the radial coordinate. It is the radial solution for a charged black hole. What is wrong with that? What we are dealing with is a frequency dependent, electromagnetic metric. It does not affect matter the way normal gravity would, but it does effect the EM waves that fall in the right bandwidth in such a way as to mimic gravity.
Todd

Offline frobnicat

  • Full Member
  • ****
  • Posts: 518
  • Liked: 500
  • Likes Given: 151
Some experimenters would be pretty content with about 100 uN but Tajmar gets about that in the direction at right angles to the frustum axis (labelled "Horizontal" on his graph). Having read through the foregoing posts, it seems likely that the huge waveguide orifice has something to do with this. He's basically extended the cavity geometry a little way at right angles to the frustum main axis.

Excellent observation, deltaMass.  In retrospect this is almost obvious, but it is not pointed out in the paper as a possible explanation for the side force.

This is another possible question to discuss with Tajmar at the AIAA presentation.

Those with access to the paper can check that maybe the vertical set-up of frustum (on horizontal balance measurement) is with the waveguide axis aligned with arm : if we define a plane with both the axis of frustum and axis of waveguide, thrust would be expected only in this plane of symmetry, if this plane is parallel to vertical axis of rotation it would come as a surprise that there is some measured thrust vector orthogonal to such plane of symmetry, by symmetry argument alone, regardless of possible mechanisms unless some very strong (and strange) kind of parity breaking, i.e. naming axis of frustum is X and waveguide is (roughly orthogonal) Y it's like measuring net thrust on Z, sorry for the clumsy wording...

Offline Rodal

  • Senior Member
  • *****
  • Posts: 5911
  • USA
  • Liked: 6124
  • Likes Given: 5564
Some experimenters would be pretty content with about 100 uN but Tajmar gets about that in the direction at right angles to the frustum axis (labelled "Horizontal" on his graph). Having read through the foregoing posts, it seems likely that the huge waveguide orifice has something to do with this. He's basically extended the cavity geometry a little way at right angles to the frustum main axis.

Excellent observation, deltaMass.  In retrospect this is almost obvious, but it is not pointed out in the paper as a possible explanation for the side force.

Is it this one?

This is another possible question to discuss with Tajmar at the AIAA presentation.

Those with access to the paper can check that maybe the vertical set-up of frustum (on horizontal balance measurement) is with the waveguide axis aligned with arm : if we define a plane with both the axis of frustum and axis of waveguide, thrust would be expected only in this plane of symmetry, if this plane is parallel to vertical axis of rotation it would come as a surprise that there is some measured thrust vector orthogonal to such plane of symmetry, by symmetry argument alone, regardless of possible mechanisms unless some very strong (and strange) kind of parity breaking, i.e. naming axis of frustum is X and waveguide is (roughly orthogonal) Y it's like measuring net thrust on Z, sorry for the clumsy wording...

Thanks, I have authorized access to the paper but I had not checked that.  What figure in the paper are you referring to ? is it this one?

I enclose strictly for discussion, research and illustration purposes Fig. 7 c of Tajmar et.al. about torsional pendulum


 “…for purposes such as criticism, comment, news reporting, scholarship, or research…” under US Fair Use

http://fairuse.stanford.edu/overview/fair-use/what-is-fair-use/

This is the  American Institute of Aeronautics and Astronautics link to Martin Tajmar's et.al. paper, that should be obtained from the American Institute of Aeronautics and Astronautics:

Direct Thrust Measurements of an EM Drive and Evaluation of Possible Side-Effects  M. Tajmar and G. Fiedler
51st AIAA/SAE/ASEE Joint Propulsion Conference

http://arc.aiaa.org/doi/pdf/10.2514/6.2015-4083
« Last Edit: 07/26/2015 06:47 pm by Rodal »

Offline X_RaY

  • Full Member
  • ****
  • Posts: 852
  • Germany
  • Liked: 1146
  • Likes Given: 2479
@WarpTech
was confused yesterday caused by the notification "kr". Dr Rodal posted the description of what is meaning it (namely not the radii of the cone at a given point at the z-axis, but the radius based on the cone apex).

@Rodal
Thanks for the explanation
http://forum.nasaspaceflight.com/index.php?topic=37642.msg1409751#msg1409751

I'm not confused by this. I know that kr = k*r, is the distance from the cone apex in terms of phase. I agree the paper is confusing in the way it is written because they also use kr.

In my equation for the tapered cylinder using Bessel functions, I had z*w/c, which is the same thing. For a given k value based on the input frequency, k being the wavenumber of the propagating mode in the z direction, r is the distance from the apex. It is simply their way of normalizing the graph, because computing the Hankle functions is easier that way.

The graphs I posted for the Reissner-Nordstrom metric are equivalent representations for k=1, where r is the radial coordinate. It is the radial solution for a charged black hole. What is wrong with that? What we are dealing with is a frequency dependent, electromagnetic metric. It does not affect matter the way normal gravity would, but it does effect the EM waves that fall in the right bandwidth in such a way as to mimic gravity.
Todd

Is your underlying idea that the expanding photon gas give some extra thrust?
May be in such a situation you described (open horn antenna?) could be more thrust like simple photon rocket.
(expanding of the photon gas plus backreaction while radiation?)
For conical cavity thrusters who are not spitz and closed to be a resonator the situation is different? I am not sure.
Some times ago i had a similar idea...
Look at point "2."
http://forum.nasaspaceflight.com/index.php?topic=37642.msg1382616#msg1382616

Or do you think this is at the end a pure gravity effect?

Offline Rodal

  • Senior Member
  • *****
  • Posts: 5911
  • USA
  • Liked: 6124
  • Likes Given: 5564
Those with access to the paper can check that maybe the vertical set-up of frustum (on horizontal balance measurement) is with the waveguide axis aligned with arm : if we define a plane with both the axis of frustum and axis of waveguide, thrust would be expected only in this plane of symmetry, if this plane is parallel to vertical axis of rotation it would come as a surprise that there is some measured thrust vector orthogonal to such plane of symmetry, by symmetry argument alone, regardless of possible mechanisms unless some very strong (and strange) kind of parity breaking, i.e. naming axis of frustum is X and waveguide is (roughly orthogonal) Y it's like measuring net thrust on Z, sorry for the clumsy wording...

Thanks, I have authorized access to the paper but I had not checked that.  What figure in the paper are you referring to ? is it this one?

I enclose strictly for discussion, research and illustration purposes Fig. 7 c of Tajmar et.al. about torsional pendulum...
http://arc.aiaa.org/doi/pdf/10.2514/6.2015-4083

Notice from the COMSOL FEA picture of the electromagnetic field in my post http://forum.nasaspaceflight.com/index.php?topic=37642.msg1409813#msg1409813

that the waveguide is NOT aligned with the center axis of axi-symmetry of the truncated cone EM Drive.

If you were to cut the EM Drive truncated cone along a plane intersecting the axis of axi-symmetry where the electromagnetic field is shown in the COMSOL image, the waveguide enters the EM Drive completely on the 1/2 of the EM Drive.  The center axis of the waveguide is way off, away from the plane where the electromagnetic field is shown.

Thus << if we define a plane with both the axis of frustum and axis of waveguide>> is not the correct plane, because the waveguide is entering the EM Drive off-axis.

Could you envision a torque, due to the waveguide entering the EM Drive not symmetrically, resulting from two thrust vectors:  one thrust vector aligned with the axis of axi-symmetry of the EM Drive and the other axis being the axis of the waveguide?

Imagine a vertical vector (due to the EM Drive cone) and a horizontal force vector that is acting at a distance R from the vertical vector.  This seems to result in a torque.  If everything is rigid, this torque is reacted by the table, and it may show up as a torsional displacement, registering as a horizontal force (which arises from the torque produced by the waveguide acting asymmetrically).
« Last Edit: 07/26/2015 07:17 pm by Rodal »

Offline TheTraveller

What I can't understand about Tajmar's build is he could have easily duplicated Shawyer's 1st Experimental EMDrive as it was driven by a non water cooled low cost 2.45GHz magnetron just as Tajmar used.

His vacuum chamber is more than large enough to handle the 160mm diameter big end.

At 16mN Force output, his measurement system could have easily measured the Force generated.

This Tajmar mini EMDrive is really a very strange build.
It Is Time For The EmDrive To Come Out Of The Shadows

Offline Rodal

  • Senior Member
  • *****
  • Posts: 5911
  • USA
  • Liked: 6124
  • Likes Given: 5564
Links can be used to refer to previous posted images.  This saves bandwidth, and prevents the thread to be clogged by pictures of the same image being posted again and again
« Last Edit: 07/26/2015 06:58 pm by Rodal »

Offline wembley

  • Member
  • Posts: 27
  • London
  • Liked: 14
  • Likes Given: 1

I'm concerned with the headline "EmDrive could reach Pluto in 18 months" being so at odds with what I heard was Prof. Tajmar's experimental data he had obtained a couple of months ago:


Well, it might be that Prof Tajmar's setup had rather a low Q value, but this was not important as his test apparatus went down to sub-micronewton measurement.
Did Tajmar ever say that his experimental results can be extrapolated to reach Pluto in 18 months using an EM Drive as the means of propulsion?
No, that's from a NASA 0.4 model

Offline TheTraveller


I'm concerned with the headline "EmDrive could reach Pluto in 18 months" being so at odds with what I heard was Prof. Tajmar's experimental data he had obtained a couple of months ago:


Well, it might be that Prof Tajmar's setup had rather a low Q value, but this was not important as his test apparatus went down to sub-micronewton measurement.
Did Tajmar ever say that his experimental results can be extrapolated to reach Pluto in 18 months using an EM Drive as the means of propulsion?
No, that's from a NASA 0.4 model

From this Eagleworks slide.

Shawyer's Flight Thruster test data does 0.4N/kW. Prof Yang has test data showing 1N/kW and 4N/kW.
« Last Edit: 07/26/2015 07:08 pm by TheTraveller »
It Is Time For The EmDrive To Come Out Of The Shadows

Offline wembley

  • Member
  • Posts: 27
  • London
  • Liked: 14
  • Likes Given: 1
With such small thrust and the apparatus not being self-contained, I am wary to break out any champagne here.

Tajmar concludes:


Quote from: Tajmar and Fiedler
The nature of the thrusts observed is still unclear. Additional tests need to be carried out to study the magnetic interaction of the power feeding lines used for the liquid metal contacts. Our test campaign can not confirm or refute the claims of the EMDrive but intends to independently assess possible side-effects in the measurements methods used so far 

This is the most lukewarm kind of support imaginable.  Shawyer is encouraged by this ?  ( http://www.wired.co.uk/news/archive/2015-07/24/emdrive-space-drive-pluto-mission  )

Quote from: Wired article The 'impossible' EmDrive could reach Pluto in 18 months
Roger Shawyer is encouraged by Tajmar's work, which he says validates his own theoretical predictions as well as his experimental results.   

The authors state that they can not confirm or refute the claims of the EMDrive     !!!!!

(page 9 of Tajmar's report)

If the authors themselves conclude that they cannot confirm or deny the EM Drive claims, there is nothing here for John Baez or Sean Carroll to have to respond to.  All that Baez and Carroll have to do is to quote the authors saying that the authors accept that their report is inconclusive !

Disagree. What Tajmar is saying in effect is
1) The device produces exactly the thrust that Shawyer predicts
2) He can't explain it
3) He can't find any faults or error in the apparatus to explain it

As far as Tajmar can tell, Shawyer is 100% right. That's pretty strong support, IMO

Offline Rodal

  • Senior Member
  • *****
  • Posts: 5911
  • USA
  • Liked: 6124
  • Likes Given: 5564
What I can't understand about Tajmar's build is he could have easily duplicated Shawyer's 1st Experimental EMDrive ...This Tajmar mini EMDrive is really a very strange build.
Tajmar's paper:

Quote
We would like to thank Roger Shawyer for his assistance 

Offline Rodal

  • Senior Member
  • *****
  • Posts: 5911
  • USA
  • Liked: 6124
  • Likes Given: 5564
...As far as Tajmar can tell, Shawyer is 100% right. That's pretty strong support, IMO

The authors of the paper (M. Tajmar and G. Fiedler) speaking in their own words:

Quote from: M. Tajmar and G. Fiedler
It must be noted that Shawyers analysis and claims are highly controversial (e.g. Ref. 9) as this would obviously violate the conservation of momentum (pushing against itself) following his theory

Quote from: M. Tajmar and G. Fiedler
Additional tests need to be carried out to study the magnetic interaction of the power feeding lines used for the liquid metal contacts. Our test campaign can not confirm or refute the claims of the EMDrive but intends to independently assess possible side-effects in the measurements methods used so far. Nevertheless, we do observe thrusts close to the magnitude of the actual predictions after eliminating many possible error sources that should warrant further investigation into the phenomena. Next steps include better magnetic shielding, further vacuum tests and improved EMDrive models with higher Q factors and electronics that allow tuning for optimal operation. As a worst case we may find how to effectively shield thrust balances from magnetic fields.

Bold added for emphasis

This is the  American Institute of Aeronautics and Astronautics link to Martin Tajmar's et.al. paper, that should be obtained from the American Institute of Aeronautics and Astronautics:

Direct Thrust Measurements of an EM Drive and Evaluation of Possible Side-Effects  M. Tajmar and G. Fiedler
51st AIAA/SAE/ASEE Joint Propulsion Conference

http://arc.aiaa.org/doi/pdf/10.2514/6.2015-4083
« Last Edit: 07/26/2015 07:19 pm by Rodal »

Offline flux_capacitor

  • Full Member
  • ****
  • Posts: 708
  • France
  • Liked: 860
  • Likes Given: 1076
Quote
Additional tests need to be carried out to study the magnetic interaction of the power feeding lines used for the liquid metal contacts. Our test campaign can not confirm or refute the claims of the EMDrive

The bold emphasis is there because of the sentence preceding it: there is a magnetic interaction of the power feeding lines used for the liquid metal contacts, which produces an anomalous thrust in the "null" vertical configuration, which was intended as the experimental control. The experimental control is screwed up. But does that mean the positive horizontal thrust in the positive direction, and the negative horizontal thrust in the negative direction, both in agreement with the theoretical predictions, are wrong?
« Last Edit: 07/26/2015 07:17 pm by flux_capacitor »

Tags:
 

Advertisement NovaTech
Advertisement Northrop Grumman
Advertisement
Advertisement Margaritaville Beach Resort South Padre Island
Advertisement Brady Kenniston
Advertisement NextSpaceflight
Advertisement Nathan Barker Photography
0